You purchase a new laptop today. The value of that laptop depreciates based on the function f(t) = 4,300(0.83)t, where t is measured in years after purchase. How much is the laptop worth after five halves years, rounded to the nearest dollar?

$2,105

$1,601

$2,699

$3,215

Answers

Answer 1

The laptop worth after five halves years will be $1,543. Then the correct option is E.

What is an exponent?

Let a be the initial value and x be the power of the exponent function and b be the increasing factor.

The exponent is given as

y = a(b)ˣ

You purchase a new laptop today. The value of that laptop depreciates based on the function is given below.

f(x) = 4,300(0.83)ˣ.

Where 'x' is the number of years.

The laptop worth after five halves years will be given as,

[tex]\rm f(x) = \$4,300 \times (0.83)^{5.5}[/tex]

f(x) = $4,300 × 0.35886

f(x) = $1,543

The laptop worth after five halves years will be $1,543. Then the correct option is E.

More about the exponent link is given below.

https://brainly.com/question/5497425

#SPJ1

The missing options are given below.

$2,105

$1,601

$2,699

$3,215

$1,543

Answer 2

The worth of the laptop after 2.5 years is $2,699.

What is a function?

function is a relationship between inputs where each input is related to exactly one output.

Example:

f(x) = 2x + 1

f(1) = 2 + 1 = 3

f(2) = 2 x 2 + 1 = 4 + 1 = 5

The outputs of the functions are 3 and 5

The inputs of the function are 1 and 2.

We have,

f(t) = 4,300 [tex](0.83)^t[/tex]

Now,

For t = 5/2,

t = 2.5

f(5) = 4300 x [tex]0.83^{2.5}[/tex]

f(5) = 2699

Thus,

The laptop is worth $2,699 after 2.5 years.

Learn more about functions here:

https://brainly.com/question/28533782

#SPJ1


Related Questions

Problem: Given an integer amount of money, you have to determinate which is the minimum amount of coins needed to arrive at the exact total. You have an infinite number of each, and may choose more than one. Coins available are $1, $5, $10, $25. Choose the highest-value coin available that does not exceed the total and repeat until you arrive at the desired amount.:

Answers

The solution is a simple process of subtracting the largest coin value from the total amount, adding any coins that fit, and repeating the process until the total amount is reached. This algorithm guarantees that the minimum number of coins needed to arrive at the total will be displayed.

1. Start by subtracting the largest coin value from the total amount.

2. If the difference is greater than 0, add the coin to the total and repeat the process.

3. If the difference is 0, move onto the next smallest coin value.

4. Repeat the process until the total amount is reached.

5. Once the total amount is reached, the minimum amount of coins needed to arrive at the total will be displayed.

The solution is a simple process of subtracting the largest coin value from the total amount, adding any coins that fit, and repeating the process until the total amount is reached. This algorithm guarantees that the minimum number of coins needed to arrive at the total will be displayed.

Learn more about minimum here

https://brainly.com/question/29156925

#SPJ4

Test market data shows that changing the price of a product changes the demand. Does this table indicate that purchase price is a function of demand? Why or why not?

Answers

The table indicates that purchase price is a function of demand, as each demand is mapped to a single purchase price.

When does a relationship represents a function?

To verify whether a relation represents a function, we need to observe if each input is mapped to only one output.

The input and output variables for this problem are given as follows:

Input: demand.Output: purchase price.

In the table, there are no repeated demand values, meaning that the purchase price is a function of the demand, as there are no inputs mapped to multiple outputs.

Missing Information

The table is given by the image presented at the end of the answer.

More can be learned about relationships and functions at brainly.com/question/29701528

#SPJ1

Suppose that x is an int variable and y is a double variable and the input is: 10 20.7 Choose the values after the following statement executes: cin >> x >> y;.a. x=10,y=20 b. x=10,y=20.0c. x=10,y=20.7 d. x=10,y=21.0

Answers

x is an int variable and y is a double variable and the input is: 10 20.7.The output will be x = 10, y = 20.7

given that

x is an int variable

y is a double variable

The C++ double should have a floating-point precision of up to 15 digits as it contains a precision that is twice the precision of the float data type.

When you declare a variable as double, you should initialize it with a decimal value. For example, 3.0 is a decimal number. However, if you initialize it with 3, it will also work fine because although 3 and 3.0 seem to be identical i.e., an integer, there is a big difference in their internal representation.

the input is: 10, 20.7

after execution of the statement

cin >> x >> y

The output will be x = 10, y = 20.7

because x is declared as integer variable so it will print the same

y is declared as double integer variable so it will print same as given input , if it is declared as integer variable then it will be printed as 20

To learn more about double variable:

https://brainly.com/question/29749526

#SPJ4

A grandmother is now 48 years older than her grandson. 14 years ago she was thirteen times as old as her grandson. How old is the grandmother? How old is the grandson?

Answers

Let's call the grandmother's current age G and the grandson's current age S.

From the first piece of information given, we know that:

G = S + 48

From the second piece of information given, we know that:

G - 14 = 13S

We can use the first equation to substitute for G in the second equation:

(S + 48) - 14 = 13S

S + 34 = 13S

34 = 12S

S = 34/12

S = 2.8 years

So the grandson is 2.8 years old.

Now we can use this information to find the grandmother's age:

G = S + 48

G = 2.8 + 48

G = 50.8 years

So, the grandmother is 50.8 years old and the grandson is 2.8 years old.

in a survey collecting data about the salaries earned by recent college graduates, li found that her salary was in the 78th percentile. that means that 78% of recent college graduates earn less than or equal to what li does. [1 point]

Answers

Li earns more than 78percentage of recent college graduates, and is in the 78th percentile of salaries earned.

1. Calculate the total number of recent college graduates surveyed: Multiply the total number of responses to the survey by 100 to get the total number of graduates surveyed.

2. Calculate the number of recent college graduates who earn less than or equal to Li's salary: Multiply the percentile (78%) by the total number of graduates surveyed to get the number of graduates who earn less than or equal to Li's salary.

3. Calculate the percentage of recent college graduates who earn less than or equal to Li's salary: Divide the number of graduates who earn less than or equal to Li's salary by the total number of graduates surveyed to get the percentage of graduates who earn less than or equal to Li's salary. The result will be 78%.

Learn more about percentage here

https://brainly.com/question/16797504

#SPJ4

answer? Drag the tiles to the correct boxes to complete the pairs.
Match each expression to its simplified form

Answers

The expression are matched to the simplest form as shown below

simplest form   Equation

12 - 1/2r              (13 - 3/2 r) - (1 - r)

r - 13/6                (7r - 3/2) - (2/3 + 6r)

13r + 20              (6r + 7) + (13 + 7r)

-12 + r                 (-8 - r) + (2r - 4)

How to find the simplest for m of each equation'

The simplest form is solved by evaluating each of the equation in the problem

Evaluating the expressions

(6r + 7) + (13 + 7r)

collecting like terms

6r + 7r + 13 + 7

13r + 20

(13 - 3/2 r) - (1 - r)

opening the parenthesis (the negative sign before the second parenthesis changes the sign in the second parenthesis)

13 - 3/2 r - 1 + r

collecting like terms

13 - 1 - 3/2 r + r

12 - 1/2 r

(-8 - r) + (2r - 4)

collecting like terms

-8 - 4 - r + 2r

-12 + r

(7r - 3/2) - (2/3 + 6r)

opening the parenthesis (the negative sign before the second parenthesis changes the sign in the second parenthesis)

7r - 3/2 - 2/3 - 6r

collecting like terms

7r - 6r - 3/2 - 2/3

r - 13/6

Learn more about simplest form of equations at:

https://brainly.com/question/26315101

#SPJ1

There are 40 female performers in a dance recital. The ratio of men to women is 5 to 8 How many men are in the dance recital?

Answers

Answer: there would be 8 men

Step-by-step explanation:

can you give me brainliest

Answer: I believe there would be 25 men

Step-by-step explanation: We can see there are 40 women and the ration is 5 (men) to 8 (women)

If we take 40 and divide it by 8 we get 5

so for every 8 women there are 5 men and we do that 5 times

we get: 40 women and 25 men.

Write the solution set in interval notation and graph the inequality.
Set-Builder Notation: {x | 5/7

Answers

The set X can therefore be written as follows: The expression "A is the set of elements x such that x is a natural number less than 7" can be translated as "A is the set of elements x such that x is a natural number less than 7." A = x: x N, x 7 is another way to express the above set.

What is the Set Builder Form or Rule Method?If a set's components share a characteristic, the characteristic can be used to define the components. For instance, the natural numbers 1, 2, 3, 4, 5, and 6 in the set A have a trait in common that all of their members fall below the number 7. This trait is not shared by any other natural numbers. The set X can therefore be written as follows:The expression "A is the set of elements x such that x is a natural number less than 7" can be translated as "A is the set of elements x such that x is a natural number less than 7."A = x: x N, x 7 is another way to express the above set.The expression "the set of all natural numbers smaller than seven" can also be used to denote set A.The common property of the elements of a set is described in this instance inside braces. A set-builder form, often known as a rule technique, takes this simple structure.

The Complete Question is  Set Builder .

To Learn more About set of elements refer to:

https://brainly.com/question/24462379

#SPJ1

Polynomial 1: 4x2
+ 5x − 4
Polynomial 2: 4x2
+ 4x − 2 What is the product of the two polynomials?
Responses


A 32x4
+ 36x3
− x2
+ 26x + 432 x 4 + 36 x 3 − x 2 + 26x + 4


B 16x4
+ 36x3
− x2
+ 26x + 416 x 4 + 36 x 3 − x 2 + 26x + 4


C 8x4
+ 18x3
− 4x2
− 26x + 88 x 4 + 18 x 3 − 4 x 2 − 26x + 8


D 16x4
+ 36x3
− 4x2
− 26x + 8

Answers

The product of the two polynomials is 16x^4 + 36x^3 - 4x^2 - 26x + 8. The solution is obtained by the method of polynomial multiplication.

What is polynomial multiplication?

The process of multiplying two or more polynomials together is known as polynomial multiplication. To obtain the final polynomial, the terms of the first and second polynomials are multiplied. We can multiply polynomials in a variety of ways depending on the kinds we are using. Each form of polynomial has a different set of guidelines for multiplication. Polynomials are multiplied by multiplying the coefficient by another coefficient and the variable by another variable.

We are given two polynomials

Polynomial 1: 4x^2 + 5x − 4

Polynomial 2: 4x^2 + 4x − 2

Multiplying both, we get

⇒(4x^2 + 5x − 4)*(4x^2 + 4x − 2)

⇒16x^4 + 16x^3 - 8x^2 + 20x^3 + 20x^2 - 10x - 16x^2 - 16x + 8

16x^4 + 36x^3 - 4x^2 - 26x + 8

Hence, the product of the two polynomials is

16x^4 + 36x^3 - 4x^2 - 26x + 8.

Learn more about polynomial multiplication from the given link

https://brainly.com/question/8690411

#SPJ1

12% of x =360 hhhhhhhhhhhhhhhhhhhhhhhhh

Answers

Answer:

x = 3000

Step-by-step explanation:

Answer:

x=3000

Step-by-step explanation:

Given that, 12% of x=360

 or, (x)× (12÷100)=360

 or, (12x ÷100)=360

 or, 12x=36000

 or, x=36000÷12

 or, x=3000

assess reasonableness a monument will be at the circumcenter of a triangular plot of land at the state capital. on the coordinate grid, the vertices of the triangular plot of land are a(1, 5) , b(7, 5) , and c(8, 0) .

Answers

Yes, this is reasonable. The circumcenter of a triangle is found by taking the average of the x-coordinates of the three points and the average of the y-coordinates of the three points, so for this triangle, the circumcenter would be at (5.67, 2.67).

The circumcenter of the triangular plot of land with vertices at (1, 5), (7, 5), and (8, 0) is at (5.67, 2.67), which is reasonable.

1. Calculate the average of the x-coordinates of the three vertices: (1 + 7 + 8) / 3 = 16 / 3 = 5.67

2. Calculate the average of the y-coordinates of the three vertices: (5 + 5 + 0) / 3 = 10 / 3 = 2.67

3. Therefore, the circumcenter of the triangle is (5.67, 2.67).

The circumcenter of the triangular plot of land with vertices at (1, 5), (7, 5), and (8, 0) is at (5.67, 2.67), which is reasonable.

Learn more about triangle here

https://brainly.com/question/2773823

#SPJ4

WHATS THE ANSWER? WILL APPRECIATE SOO MUCH !!

Answers

(s+12)^3

I am wasting space here as you can tell but no fr the answer is (s+12)^3

In a class of 40 students, 8 are in the drama club, 12 are in the art club, and 5 are in the music club. If a student is selected at random, what is the probability that the selected student is in none of the clubs listed?

Answers

Considering the definition of probability, the probability that the selected student is in none of the clubs listed is 0.375 or 37.5%.

Definition of probability

Probability is the greater or lesser chance that a given event will occur.

In other words, the probability establishes a relationship between the number of favorable events (number of cases in which event A may or may not occur)  and the total number of possible events through the Laplace's Law as follow:

P(A)= number of favorable cases÷number of total cases

Probability that the selected student is in none of the clubs listed

In this case, you know:

Total number of students = 40 (number of possible cases)Total number of students in drama club= 8Total number of students in art club= 12Total number of students in music club= 5Total number of students in a club= 8 + 12 + 5= 25Total number of students in none of the clubs listed= 40 - 25= 15 (number of favorable cases)

Replacing in the definition of probability:

P(A)= 15÷40

Solving:

P(A)= 0.375

Expressed as a percentage:

P(A)= 37.5%

Finally, the probability is 0.375 or 37.5%.

Learn more about probability:

brainly.com/question/3283947

#SPJ1

Find the sum and product of Eigen value of matrix A = 3 1 4 0 2 6 0 0 5

Answers

The sum of the eigenvalues of matrix A is 8 and the product of eigenvalues is -15.

What is the eigenvalue?

Eigenvalues are the special set of scalar values that are associated with the set of linear equations most probably in the matrix equations. The eigenvectors are also termed characteristic roots.

To find the eigenvalues of matrix A, we can solve the characteristic equation |A - λI| = 0, where λ is a scalar and I is the identity matrix.

|A - λI| = 3- λ 1 4 0 2- λ 6 0 0 5- λ

= (3-λ)(5-λ) - 24 = λ^2 - 8λ - 15

Setting λ^2 - 8λ - 15 = 0, we can find the eigenvalues of the matrix by solving the quadratic equation.

λ^2 - 8λ - 15 = 0

λ = (8 ± √(8^2 + 4*15))/2

λ = (8 ± √(64 + 60))/2

λ = (8 ± √124)/2

So the eigenvalues of matrix A are λ_1 = (8 + √124)/2 and λ_2 = (8 - √124)/2

The sum of eigenvalues is λ_1 + λ_2 = (8 + √124)/2 + (8 - √124)/2 = 8

The product of eigenvalues is λ_1 * λ_2 = (8 + √124)/2 * (8 - √124)/2 = -15

Hence, the sum of the eigenvalues of matrix A is 8 and the product of eigenvalues is -15.

To learn more about the eigenvalues visit,

https://brainly.com/question/15586347

#SPJ4

PLS HELP TYY!!The diagram below shows the dimensions of a can of beans.

A can of beans is shown. A dashed line drawn across the top of the can is labeled seven centimeters. The height of the can is labeled eleven and six tenths centimeters.

How much tin was used to make the can rounded to the nearest square centimeter? Use 3.14 for π.

Enter the correct answer in the box.

Answers

Answer:

  332 cm²

Step-by-step explanation:

You want to know the surface area of a tin can that is 7 cm in diameter and 11.6 cm high.

Surface area

The area of a cylinder is given by the formula ...

  SA = πd(d/2 +h) . . . . . where d is the diameter and h is the height

Application

Using the given values, d=7 and h=11.6, we find the area to be ...

  SA = (3.14)(7 cm)(3.5 +11.6 cm) ≈ 332 cm²

About 332 square centimeters of tin are required to make the can.

The number of miles y is proportional to the number of hours x as shown in the graph.

Answers

An equation to represent this proportional relationship is y = 65x.

What is the point-slope form?

Mathematically, the point-slope form of a straight line can be calculated by using this mathematical expression:

y - y₁ = m(x - x₁) or y - y₁ = (y₂ - y₁)/(x₂ - x₁)(x - x₁)

Where:

y represents the number of hoursx represents the number of miles.m represents the slope.

Next, we would determine the linear equation representing the line which passes through the points (3, 195) and (6, 390) by using the point-slope form as follows:

y - y₁ = (y₂ - y₁)/(x₂ - x₁)(x - x₁)

y - 195 = (390 - 195)/(6 - 3)(x - 3)

y - 195 = 195/3(x - 3)

y - 195 = 65(x - 3)

y = 65x - 195 + 195

y = 65x

Read more on slope here: brainly.com/question/23086745

#SPJ1

Find the general solution of the first-order linear differential equation y' - (ln x)y = 3 x^x.
y(x)=
Note: Use C for the arbitrary constant.

Answers

The general solution of the first-order linear differential equation y' - (ln x)y = [tex]3 x^x[/tex] is [tex]y=3x^{x} + cx^{x} e^{-x}[/tex]

As per the give,n data the given differential equation is:

y' - (ln x) y = 3 [tex]x^x[/tex]

Here have to determine the general solution of the first-order linear differential equation.

The differential equation is linear if the dependent variable y and its derivative only occur in the first degree.

Then, [tex]$\frac{d y}{d x}-(\ln x) y=3 x^x$[/tex]

This is of the form [tex]$\frac{d y}{d x}[/tex] + P(x)y = Q(x) which is the linear equation.

In order to solve this kind of differential equation, we have to multiply it by an integrating factor.

Here, P = - ln (x), Q(x) = 3 [tex]x^x[/tex]

I F = [tex]e^{\int P d x}[/tex]

I F = [tex]e^{-\int \ln (x)}[/tex]

I F = [tex]e^{-(x \ln x-x)}[/tex]

I F = [tex]e^{x-x \ln x}[/tex]

I F = [tex]\frac{e^x}{e^{x \ln x}}[/tex]

I F = [tex]\frac{e^x}{x^x}[/tex]

Hence, the general solution is

[tex]& y(I F)=\int Q(x)(I F) d x+c \\[/tex]

[tex]& \Rightarrow y\left(\frac{e^x}{x^x}\right)=\int 3 x^x\left(\frac{e^x}{x^x}\right) d x+c \\[/tex]

[tex]& \Rightarrow y\left(\frac{e^x}{x^x}\right)=\int 3 e^x d x+c \\[/tex]

[tex]& \Rightarrow y\left(\frac{e^x}{x^x}\right)=3 e^x+c \\[/tex]

[tex]& \Rightarrow y=3 x^x+c x^x e^{-x}[/tex]

Hence, the general solution is [tex]y=3x^{x} + cx^{x} e^{-x}[/tex]

For more questions on linear differential equations

https://brainly.com/question/14865813

#SPJ4

I WILL GIVE BRAINLIEST!!!!
To make 4 cups of punch, David needs 3 cups of cranberry juice. David wants to make 72 cups of punch.


How many cups of cranberry juice will he need?

Answers

Answer:   54 cups of cranberry juice

Work Shown:

4/3 = 72/x

4x = 3*72

4x = 216

x = 216/4

x = 54

Explanation:

Each fraction in the first step is of the form A/B, where A is the amount of punch and B is the amount of cranberry juice. Both measured in cups. In the 2nd step, I cross multiplied. Afterward, divide both sides by 4 to isolate x.

Which graph represents a proportional​ relationship? Explain

Answers

Graph A.

Proportional relationships have to go through the origin, (0,0).

a 2.5 nm torque is applied to a flywheel for 10.0 seconds. if the flywheel is initially at rest and its moment of inertia is 2.0 kg m2, determine its angular momentum. group of answer choices

Answers

The angular momentum of the flywheel is equal to the product of the applied torque, the time interval, and the moment of inertia. Therefore, the angular momentum of the flywheel will be 2.5 Nm × 10.0 s × 2.0 kg m2 = 50 kg m2/s.

1. Determine the angular momentum of the flywheel:

Angular momentum = Torque × Time × Moment of Inertia

= 2.5 Nm × 10.0 s × 2.0 kg m2

= 50 kg m2/s

The angular momentum of an object can be determined from the product of the applied torque, the time interval, and the moment of inertia. The moment of inertia is the rotational inertia of an object, which is the resistance to changes in its angular velocity. The angular momentum of the flywheel in this question is equal to the product of the applied torque, the time interval, and the moment of inertia. Therefore, the angular momentum of the flywheel will be 2.5 Nm × 10.0 s × 2.0 kg m2 = 50 kg m2/s. This means that the flywheel will have an angular momentum of 50 kg m2/s after the 10-second interval when the torque is applied. The angular momentum of the flywheel will remain constant until another torque is applied to change it.

Learn more about interval here

https://brainly.com/question/24131141

#SPJ4

1. Jeremy's father is paying for a $30 meal. He has a 20% off coupon for the meal. How much money will the coupon save Jeremy's dad?
2. Jeremy's father is paying for a $30 meal. He has a 20% off coupon for the meal. After the discount, a 6% sales tax is applied. How much will Jeremy's father pay in sales tax?
3. Jeremy's father is paying for a $30 meal. He has a 20% off coupon for the meal. How much in total did Jeremy's father pay for the meal (including tax)?

Answers

1. The coupon will save: $6.

2. The sales tax is of: $1.44.

3. The total price paid for the meal is of: $25.44.

How to obtain the total cost?

The total cost is obtained applying the proportions in the context of this problem.

The initial fee for the meal is of:

$30.

With the coupon of 20% off, the amount paid is of:

0.8 x 30 = $24.

Meaning that the amount saved is of:

30 - 24 = $6.

After the sales tax, the total price is of:

1.06 x 24 = $25.44.

Meaning that the tax is of $1.44.

More can be learned about proportions at https://brainly.com/question/24372153

#SPJ1

I need help with the question below

Answers

Answer:

see below

Step-by-step explanation:

there are 3 points given

diff in y / diff in x = slope

so plug in any 2 points' Y and x and get the slope

because it's downward shaped, you  know the slope MUST be negative.

so diff in Y = 23-29 = -6

diff in X = 3-7 =2

so slope is +3

intercept is given, when x=0 y = 32

it represents what F is when it's 0C°

If y varies directly as x and the constant of variation is -2, which equation represents this relationship

Answers

The equation represents this relationship between the variables 'x' and 'y' will be y = -2x.

What are ratio and proportion?

A ratio is a group of sequentially ordered numbers a and b expressed as a/b, where b is never equal to zero. When two objects are equal, a statement is said to be proportional.

If y varies directly from x. Then the proportionality is given as,

y ∝ x

If the proportionality signature is removed then a constant is introduced. Then the equation is given as,

y ∝ x

y = kx

The constant of variation is -2. Then the equation is given as,

y = - 2x

The equation represents this relationship between the variables 'x' and 'y' will be y = -2x.

More about the ratio and the proportion link is given below.

https://brainly.com/question/14335762

#SPJ1

You roll a 6-sided die.
What is P(greater than 5 or less than 3)?
2/6
2/3
1/2
1/6

Answers

1/6

The probability of an event is the number of ways the event can happen divided by the number of possible outcomes. In this case, the possible outcomes are the numbers on the die, which are 1, 2, 3, 4, 5, and 6.

The event of rolling a number greater than 5 or less than 3 can happen in two ways: rolling a 6 or rolling a 1 or 2. So the probability of this event is:

P(greater than 5 or less than 3) = (number of ways the event can happen) / (number of possible outcomes) = 2/6 = 1/3

So the answer is option D. 1/6

Describe how you can determine the domain and range of f(x) = −8x7 + 1.9x6 − 7x5 + 0.12x3 + 22x2 + 4.

Answers

The domain of a function is the set of possible values that can be plugged into it.

What is the Domain and Range of a Function?

The range of values that can be plugged into a function is known as its domain. In a function like f, this set represents the x values (x). The collection of values that a function can take on is known as its range. The values that the function outputs when we enter an x value are in this set.

The collection of all possible inputs and outputs constitutes a function's domain and range, respectively. Important features of a function are the domain and range. The domain accepts all potential input values from the set of real numbers, and the range accepts all function output values.

To learn more about Function refer to:

https://brainly.com/question/11624077

#SPJ1

Select all the correct locations on the image. Identify which functions have complex roots by selecting the function names on the provided coordinate plane.

Answers

Out of the given functions, a ,b, c, d , the function with complex roots are b and d.

What are complex roots?

Complex roots, also known as non-real roots, are roots of an equation that are not real numbers but instead are complex numbers. Complex roots are usually represented in the form of a + bi, where a and b are real numbers and i is the imaginary unit, which is defined as the square root of -1.

The functions which have complex roots are b and d.

hence, out of the given functions, a ,b, c, d , the function with complex roots are b and d.

To learn more about the complex roots, visit:

https://brainly.com/question/25841119

#SPJ4

aisha, benoit, and carleen are each thinking of a positive integer. aisha's number and benoit's number have a common divisor greater than 1. aisha's number and carleen's number also have a common divisor greater than 1. benoit's number and carleen's number also have a common divisor greater than 1. is it necessarily true that the greatest common divisor of all three numbers is greater than 1? if it is necessarily true, explain how you know it's true. if it isn't necessarily true, convince us by showing us three numbers for which it's false -- in other words, show us three integers such that every pair have a common divisor greater than 1, but all three numbers don't have a common divisor greater than 1. hint(s):

Answers

No, it is not true that GCD(A, B, C).

What is the greatest common divisor?

The greatest common factor (GCF) that divides two or more numbers is known as the greatest common divisor (GCD). The highest common factor is another name for it (HCF). For instance, since both 15 and 10 can be divided by 5, 5 is the biggest common factor between both.

Here, we have

Given: Aisha's number and Benoit's number have a common divisor greater than 1. Aisha's number and Carleen's number also have a common divisor greater than 1. Benoit's number and Carleen's number also have a common divisor greater than 1.

We have to determine if is it necessarily true that the greatest common divisor of all three numbers is greater than 1.

Here, we solve this with an example.

A = 6 = 2*3,

B = 14 = 2*7,

C = 21 = 3*7

There is no GCD greater than 1 that divides all of A, B and C.

Hence, No, it is not true that GCD(A, B, C).

To learn more about the greatest common divisor from the given link

https://brainly.com/question/29514635

#SPJ4

After giving a test to a group of students, below is the summary of grades by gender. A B C Total Male 16 17 11 44 Female 14 6 3 23 Total 30 23 14 67 If a student is randomly chosen, find the probability that the student is female or earned grade C. Leave your answer as a reduced fraction.

Answers

The probability that the student is female or earned grade C is 0.6

How to find the probability?

You should recall that probability is the chance of occurrence of an event E

P(E) = (Number of required outcome)/(number of possible outcomes)

The given parameters are

We'll use this table (below) to answer the questions

                         A B C Total

Male           16 17 11 44

Female           14 6 3 23

Total          30 23 14 67

Total number of female = 23Total number in grace C = 14Total number of students = 67

Pr(E) = 23/67 + 14/67

Simplifying the fractions to have

37/67 = 0.55

Therefore this is approximately rounded to 1 decimal place as  0.6

Learn more about probabilities on https://brainly.com/question/30034780

#SPJ1

One numbe is 5 times a first number. A third number is 100 more than the first number. If the sum of the three numbers is 513 . Find the numbers

Answers

Answer:

x = 56

First number = 56

Second number = 280

Third number = 156

Total 331

Step-by-step explanation:

Input (x) 1 2 4 5
Output (y) 6 12 18 24 30
1
6 12
2 3
18
4
24
5
30

Answers

The output is determined by the pattern of the input.

What is pattern?

Pattern is a repeated design or motif used to create a visually pleasing effect. Patterns are often seen in nature, architecture, clothing, and artwork. Patterns can be used to create shapes, textures, and decorations. They can add interest and depth to a design. Patterns can be made up of geometric shapes, lines, curves, and combinations of those elements. They can be created with a variety of materials and techniques, including weaving, printing, embroidery, and painting. Patterns can be used to create continuity, emphasize a focal point, or provide a sense of balance. Patterns can be used to draw the eye to a particular area, or to create a sense of order and organization.

In this example, each output number is the sum of the two numbers before it. Therefore, the output is 6 (1 + 2), 12 (2 + 4), 18 (4 + 5), 24 (5 + 6), and 30 (6 + 8).

To learn more about pattern
https://brainly.com/question/27033681
#SPJ1

Other Questions
the oldest known mention of grass in literature is in the koran.a. trueb. false Write a function rule for the area of a triangle whose base is 4 ft more than the height. What is the area of the triangle when its height is 6 ft? A. A = 0.5h2 + 4h; 42 ft2 B. A = 4h 0.5h2; 6 ft2 C. A = 0.5h2 + 2h; 30 ft2 D. A = 0.5h2 2h; 6 ft2 In Java: Write a program whose inputs are three integers, and whose output is the smallest of the three values.Ex: If the input is: 7 15 3Output: 3Code:import java.util.Scanner;public class LabProgram{public static int LargestNumber(int num1, int num2, int num3){if(num1 > num2 && num1 > num3)return num1;else if(num2 > num3)return num2;elsereturn num3;}public static int SmallestNumber(int num1, int num2, int num3){if(num1 < num2 && num1 < num3);return num1;else if(num2 < num3)return num2;elsereturn num3;}public static void main(String[] args){int a,b,c, largest, smallest;Scanner sc=new Scanner(System.in);System.out.println("\nEnter the numbers:");a=sc.nextInt();b=sc.nextInt();c=sc.nextInt();largest = LargestNumber(a,b,c);smallest = SmallestNumber(a,b,c);System.out.println("\nLargest: "+largest);System.out.println("Smallest: "+smallest);}}Error: LabProgram.java:20: error: 'else' without 'if' else if(num2 < num3) e ksa methods of involving clients/client systems in problem identification (e.g., gathering collateral information Without using calculator, determine between which two integers the following surds lie (d)30 (e)45 identifying and defining for the calendar year is essential when planning your social media activity. Explain why COVID is not classified like the cells it infects The thousands of Northerners who traveled to the south after the war to impose their version of a punitive Reconstruction on the defeated southerners saw the south as a ________. ASAP: ANSWERCan someone solve the question in the picture? What is blank verse how does it differ from a sonnet?. What is perfect number class 6?. which of the given purposes is the tlc method most often used for? Help stuck on this one question What is first stage direction?. Nitrogen's electronegativity value is between those of phosphorus and oxygen.Which of the following correctly describes the relationship between the three values?A. The value for nitrogen is less than that of phosphorus because nitrogen is larger, but greater than that of oxygen because nitrogen has a greater effective nuclear charge.B. The value for nitrogen is less than that of phosphorus because nitrogen has fewer protons but greater than that of oxygen because nitrogen has less valence electrons.C. The value for nitrogen is greater than that of phosphorus because nitrogen has less electrons, but less than that of oxygen because nitrogen is smaller.D. The value for nitrogen is greater than that of phosphorus because nitrogen is smaller, but less than that of oxygen because nitrogen has a smaller effective nuclear charge. on which of the following issues were delegates able to overcome conflict and reach a compromise during the constitutional convention? Compare and Contrast the exploration efforts of the Spanish, French, Dutch, and English. What were their motives being there? Why were the English more successful in their settlement of the New World? Susan is the manager of a Burger King store. Which of the following actions would lead you to believe that Susan is not a modern manager?Susan trains her employees in customer service and tells them that they can decide how to resolve customer complaints. Susan tells her employees they will receive bonuses if they do their work according to company guidelines Susan asks her employees to work as a collaborative team, rather than being individual producers, Susan provides her employees with the resources they need to do their jobs effectively, Which Of the following could be the ratio between Of the two legs Of a 30-60-90 triangle? (Apex Quiz) Sal's Sandwich Shop sells wraps and sandwiches as part of its lunch specials. The profit on every sandwich is $2, and the profit on every wrap is $3. Sal made a profit of $1,470 from lunch specials last month. The equation 2x + 3y = 1,470 represents Sal's profits last month, where x is the number of sandwich lunch specials sold and y is the number of wrap lunch specials sold. Describe how you would graph this line using the slope-intercept method. Be sure to write using complete sentences. Write the equation in function notation. Explain what the graph of the function represents. Be sure to use complete sentences. Graph the function. On the graph, make sure to label the intercepts. You may graph your equation by hand on a piece of paper and scan your work or you may use graphing technology. Suppose Sal's total profit on lunch specials for the next month is $1,593. The profit amounts are the same: $2 for each sandwich and $3 for each wrap. In a paragraph of at least three complete sentences, explain how the graphs of the functions for the two months are similar and how they are different.